LSAT and Law School Admissions Forum

Get expert LSAT preparation and law school admissions advice from PowerScore Test Preparation.

 LSAT2018
  • Posts: 242
  • Joined: Jan 10, 2018
|
#48002
I somehow thought of this as a Mistaken Reversal (the second and third sentences in the stimulus) and chose answer (C). Why is this approach wrong?

And as for (E), when it says 'the likelihood that a designated event in that set will occur' wouldn't the part of the premise that 'the likelihood of his winning one of them would be greater than if he entered only a single raffle' be inclusive of winning the Mayfield raffle?
User avatar
 Dave Killoran
PowerScore Staff
  • PowerScore Staff
  • Posts: 5852
  • Joined: Mar 25, 2011
|
#48227
Hi LSAT,

Ok, in sentences 2 and 3, that is a form of restatement that's occurring. This is how it looks:

  • Sentence 2: If he enters more than one raffle next year :arrow: the likelihood of his winning one of them would be greater

    Sentence 3: if you want the necessary ("have this greater likelihood of winning"), then you should do the sufficient ("John should enter several other raffles next year").
That's actually a valid thing to say (substitute in "Study" and "A+," for example, to see another example of how makes sense). So, the moment that relationship is shown as valid, you know that's not the error, and instead you are looking for some other problem. The error in non-LSAT terms is that sure, entering more raffles increases the likelihood that you'll win one of them, but that by itself doesn't mean you've increased your chances of winning the Mayfield raffle. He might instead win a different raffle, which then doesn't meet his goal. In short, entering more raffles doesn't affect his chances in the Mayfield raffle, which is separate. that's what (E) is saying :-D

And remember, this is a Flaw question, so you are describing what happened in the argument. There's a definite confusion over how the math behind the winning one in a series vs winning a single event works.

Thanks!
 LSAT2018
  • Posts: 242
  • Joined: Jan 10, 2018
|
#48786
Thank you for the explanation!
I think I mistook this for a conditional reasoning statement, but it actually isn't because of the 'greater' part. So this is more of a hypothetical causal reasoning statement.

Get the most out of your LSAT Prep Plus subscription.

Analyze and track your performance with our Testing and Analytics Package.